Difference between revisions of "2022 AMC 10A Problems/Problem 20"

(Solution)
(Improved variable definitions. Also, considered casework and denied one case.)
Line 6: Line 6:
  
 
==Solution==
 
==Solution==
Set up a system of equations.
+
Let the arithmetic sequence be <math>a,a+d,a+2d,a+3d</math> and the geometric sequence be <math>b,br,br^2,br^3.</math>
<cmath>\begin{align*}
 
a+b&=57\\
 
(a+n)+bm&=60\\
 
(a+2n)+bm^2&=91
 
\end{align*}</cmath>
 
  
Subtract the two consecutive equations to get
+
We are given that
 
<cmath>\begin{align*}
 
<cmath>\begin{align*}
b(m-1)+n&=3\\
+
a+b&=57, \\
bm(m-1)+n&=31
+
a+d+br&=60, \\
 +
a+2d+br^2&=91,
 
\end{align*}</cmath>
 
\end{align*}</cmath>
 +
and we wish to find <math>a+3d+br^3.</math>
  
Subtract those to get
+
Subtracting the first equation from the second and the second equation from the third, we get
<cmath>b(m-1)^2=28</cmath>
 
 
 
Note that the only square with integer <math>m</math> that fits the factors of <math>28</math> is <math>4.</math> Thus, we have that
 
 
<cmath>\begin{align*}
 
<cmath>\begin{align*}
m&=3 \\
+
d+b(r-1)&=3, \\
b&=7
+
d+br(r-1)&=31.
 
\end{align*}</cmath>
 
\end{align*}</cmath>
 +
Subtract these results, we get <cmath>b(r-1)^2=28.</cmath>
  
Then, <math>a=50</math> and all the known values in the second equation to get
+
Note that <math>r=2</math> or <math>b=3.</math> We proceed with casework:
 
 
<cmath>50+n+21=60</cmath>  
 
  
Thus, <math>n=-11.</math>
+
* If <math>r=2,</math> then <math>b=28,a=29,</math> and <math>d=25.</math> The arithmetic sequence is <math>29,4,-21,-46,</math> arriving at a contradiction.
  
Thus we conclude the answer to be
+
* If <math>r=3,</math> then <math>b=7,a=50,</math> and <math>d=-11.</math> The arithmetic sequence is <math>50,39,28,17,</math> and the geometric sequence is <math>7,21,63,189.</math> The answer is <math>a+3d+br^3=17+189=\boxed{\textbf{(E) } 206}.</math>
<cmath>\begin{align*}
 
(a+3n)+bm^3&=(50-33)+7 \cdot 3^3 \\
 
&= \boxed{206}
 
\end{align*}</cmath>
 
  
+mathboy282
+
~mathboy282 ~MRENTHUSIASM
  
 
== Video Solution by OmegaLearn ==
 
== Video Solution by OmegaLearn ==

Revision as of 08:16, 19 November 2022

Problem

A four-term sequence is formed by adding each term of a four-term arithmetic sequence of positive integers to the corresponding term of a four-term geometric sequence of positive integers. The first three terms of the resulting four-term sequence are $57$, $60$, and $91$. What is the fourth term of this sequence?

$\textbf{(A) } 190 \qquad \textbf{(B) } 194 \qquad \textbf{(C) } 198 \qquad \textbf{(D) } 202 \qquad \textbf{(E) } 206$

Solution

Let the arithmetic sequence be $a,a+d,a+2d,a+3d$ and the geometric sequence be $b,br,br^2,br^3.$

We are given that \begin{align*} a+b&=57, \\ a+d+br&=60, \\ a+2d+br^2&=91, \end{align*} and we wish to find $a+3d+br^3.$

Subtracting the first equation from the second and the second equation from the third, we get \begin{align*} d+b(r-1)&=3, \\ d+br(r-1)&=31. \end{align*} Subtract these results, we get \[b(r-1)^2=28.\]

Note that $r=2$ or $b=3.$ We proceed with casework:

  • If $r=2,$ then $b=28,a=29,$ and $d=25.$ The arithmetic sequence is $29,4,-21,-46,$ arriving at a contradiction.
  • If $r=3,$ then $b=7,a=50,$ and $d=-11.$ The arithmetic sequence is $50,39,28,17,$ and the geometric sequence is $7,21,63,189.$ The answer is $a+3d+br^3=17+189=\boxed{\textbf{(E) } 206}.$

~mathboy282 ~MRENTHUSIASM

Video Solution by OmegaLearn

https://youtu.be/DBHhSX8oVME

~ pi_is_3.14

See Also

2022 AMC 10A (ProblemsAnswer KeyResources)
Preceded by
Problem 19
Followed by
Problem 21
1 2 3 4 5 6 7 8 9 10 11 12 13 14 15 16 17 18 19 20 21 22 23 24 25
All AMC 10 Problems and Solutions

The problems on this page are copyrighted by the Mathematical Association of America's American Mathematics Competitions. AMC logo.png